This is an EXCEPT resolve, reconcile, explain question, indicated in the question stem by: Each of the following, if true, helps resolve the apparent discrepancy EXCEPT:
The stimulus tells us that on average people spend significantly less time reading now than 50 years ago, but many more books are bought overall. There is an intuitive discrepancy between a decrease in reading and increase in books sold, but we should recognize that there are a lot of ways of explaining it. For example, we are comparing an average to a total; if the population has increased, then we would expect more books to be sold even if the rate at which people read them, and hence bought them, decreased. That also leads to another obvious issue; can we assume people buying books means they read them? If you are anything like me, you are much better at buying books than reading them. Maybe people just have more income to spend so they are buying more books, even though they don’t have as much time to read! Since this is an EXCEPT question, we are looking for the one answer that doesn’t explain the apparent discrepancy. Let’s see what we get!
Answer Choice (A) This is one issue we identified in the stimulus; more people means more books sold, even if, on average, people read less.
Answer Choice (B) Ok, so maybe people are reading less but a larger share of readers purchase their books rather than borrow them. Makes sense!
Answer Choice (C) So a subset of literate people have a reason to purchase more books even if the set as a whole is reading less.
Correct Answer Choice (D) What people do with their books doesn’t really help us if it doesn’t explain how sales could be higher while reading declined
Answer Choice (E) If books are shorter, then people could be reading less while buying more books!
This is an EXCEPT must be true question, since the stem asks: If the statements above are true, each of the following must also be true on the basis of them EXCEPT:
Our stimulus begins with a some statement, and informs us that some of the world’s most beautiful cats are Persian cats. It’s important to remember that just because some beautiful cats are Persian, that doesn’t mean all Persian cats are beautiful. What we do learn about all Persian cats however, is that they are pompous. I’m not quite sure what it means for a cat to be pompous (I’m picturing a cat turning up its nose at dry food), but we also learn that pompous cats are invariably irritating, which means that pompous cats are always irritating. Because this is an EXCEPT must be true question, we know we are going to get four answers that are guaranteed to be true from this cat info, and that the correct answer will be the one answer which could be false. Let’s see what cat inferences our answer choices make:
Answer Choice (A) If all Persian cats are irritating, and some of the most beautiful cats are Persian, then it must be true that some of the most beautiful cats are irritating.
Answer Choice (B) Same as A, this is guaranteed
Answer Choice (C) All Persian cats are Irritating means that Persian → Irritating. C is just the contrapositive of this conditional, and therefore must be true.
Answer Choice (D) If all Persian cats are pompous, and some of the most beautiful cats are Persian, then it must be true that some of the most beautiful cats are pompous.
Correct Answer Choice (E) This could be false! For all we know all cats that are both irritating and beautiful are Persian.
We should recognize this as a weakening question: Which one of the following, if true, most seriously weakens the representative’s argument?
Our stimulus is attributed to a Beverage company representative, and he makes a prediction about the effect of a new policy. Thus far, beverage cans have been linked by plastic rings that often harm and suffocate animals when they end up in the environment. All the beverage companies, following the representative’s company’s lead, are going to adopt new plastic rings that disintegrate after three days exposure to sunlight. Sounds like a much better option! The representative concludes with a prediction that once the switchover from the old to the new plastic rings is completed, the threat of suffocation to animals will be eliminated. Our job is to weaken this prediction: we want an answer choice that would reduce the probability that the change, once adopted, will be sufficient to eliminate the threat to animals. On to the answers:
Answer Choice (A) Remember, our representative’s prediction specifies that the elimination will occur once the switchover is complete.
Correct Answer Choice (B) If the original problem is still out there, well then even if it is no longer going to be getting worse a switchover won’t be enough to eliminate the threat of plastic rings to animals.
Answer Choice (C) This doesn’t weaken the prediction at all.
Answer Choice (D) Good for the new rings, but this does nothing to weaken the prediction.
Answer Choice (E) Okay this gives us a reason why the change will harm animals, but remember, we are specifically interested in whether it will eliminate the problem of suffocating.
Here we have a weakening question, as we are introducing a premise which weakens an argument: Which one of the following, if true, could Mark cite to counter evidence offered by Tina?
Our stimulus takes the form of a debate between Mark and Tina about whether paper or plastic-foam cups are more environmentally friendly. Mark begins by telling us that p-f cups contain chlorofluorocarbons, which are bad for the environment, and gives us his conclusion; paper cups are the better option. The LSAT sure loves these tongue-twister chemical compounds! But that’s not all Mark has for us, he further informs us that the production of p-f cups also produces the carcinogen styrene, and the cups never biodegrade. They definitely don’t sound like a great option so far. Let’s see what Tina has to say!
Tina begins by claiming that Mark isn’t properly considering the downsides of paper cups. To be fair to Tina, Mark didn’t really give us an argument about why paper cups are good, he just brought up a bunch of downsides of p-f cups. Tina cites a study from 5 years earlier which concluded that paper cups required much more resources to produce than p-f cups. Even worse, paper cups take more energy to transport. And if that wasn’t enough, the paper mills produce pollution and the cups themselves produce even more when they decay. Wow, this seems like a lose-lose situation! Maybe we should all just switch to glasses? But our job is to weaken Tina’s argument; we want some evidence that will make paper cups look better than p-f cups. Let’s see what the answer choices give us:
Answer Choice (A) This just gives another downside for paper cups!
Answer Choice (B) Our argument is about the actual environmental downsides of the two cup types, and regardless this just supports Tina’s foam cups.
Correct Answer Choice (C) This is the only answer that weakens Tina’s support. If the paper mills use waste wood instead of petroleum, then Tina’s points about the relative resource consumption and mill pollution are less of an issue for Mark.
Answer Choice (D) This weakens Mark’s argument!
Answer Choice (E) Same as A!
This is a strengthening question, though it may be difficult to identify. We should know it is a strengthening question because the stem asks: On the basis of the premises advanced, which one of the following principles, if established, would provide the most justification for the concluding recommendation?
The stimulus begins with the statistic that 99% of the burglar alarms police respond to are false alarms. Seems like a big waste of time! The stimulus explicitly tells us that this is a drain on important resources, with each false alarm wasting an average of ~45 minutes of police time. This draws police attention away from other crimes and mostly benefits businesses and the rich, but on the other hand alarm systems are effective deterrents of burglaries. The author concludes by recommending a compromise: police keep responding to burglar alarms but fine alarm owners the cost for false alarms. Our job is to find the principle in the answer choices will justify the author’s recommendation. Let’s see our options:
Answer Choice (A) This principle would justify eliminating burglar alarm systems, not the author’s compromise.
Answer Choice (B) The author is recommending that burglar systems owners be fined in specifically the case of false alarms, regardless of whether police resources are in short supply or they can afford to.
Answer Choice (C) We haven’t been told anything about whether the author’s recommendation would reduce crime levels throughout the entire area the police serve, just that they are deterrent for their owners!
Answer Choice (D) Important to remember the condition for people being required to pay is if they waste police time with a false alarm, not whether they directly benefit from the police service
Correct Answer Choice (E) We’ve been explicitly told that false alarms waste police time which could be spent on other legitimate calls, so this would justify the author’s recommendation that false alarm owners be fined.
We should recognize this as a must be true question, as it asks: If both G’s assertions and H’s assertions are true, which one of the following must also be true*?*
This MBT question takes the form of a dialogue between “G” and “H”. G thinks the Met art show is biased because more photographs were shown despite equal submissions between photographs, sculptures, and paintings. So it seems like photographs were selected at higher rates despite applying at the same rate. H counters this accusation of bias by informing us that all and only those works that met the traditional criteria were submitted. An all and only statement indicates a biconditional, in this case: met criteria ←→ exhibited. Every application that met the criteria was exhibited, and every exhibited artwork met the criteria. In that case it seems like, rather than bias, more sculptures and paintings just didn’t meet the traditional criteria this year. Since the criteria is traditional, we can infer it is the same as prior years, and therefore doesn’t represent a change that could be biased. Let’s see the answer choices:
Answer Choice (A) This must be false as we were told there were equal numbers of submissions between categories, and each artist was only allowed to submit work in one category.
Answer Choice (B) All the exhibited artworks did, but since all the artworks that met the criteria were exhibitted, and not all submitted works were exhibited, then some submitted works did not meet the traditional criteria.
Answer Choice (C) We are told nothing about comparative quality.
Answer Choice (D) For all we know all submitted photographs were accepted.
Correct Answer Choice (E) Bingo! Based on our biconditional and the fact that more photographs were exhibited, more photograph submissions must have met the traditional criteria.
This is a must be true question, as it asks: If the statements above are all true, which one of the following must also be true on the basis of them?
We learn that Jennifer, an employee of three years, is entitled to four weeks of paid vacation and plans to spend all of it on vacation with her family. Good for Jennifer! The next sentence tells us that anyone who has worked between one and four years is automatically entitled to three weeks paid vacation a year but can apply up to half of any vacation time leftover in a year to the next year’s vacation. This must apply to Jennifer, so three weeks of her four weeks vacation time must be because she has worked between one and four years. An important word in this stimulus is exactly. From this we can infer that this three weeks is all the entitled vacation time workers get excepting from applying leftover time, and therefore if Jennifer has four weeks, it must because she had leftover vacation time from last year. And that’s all the information we get! On to the answers:
Correct Answer Choice (A) This is exactly what we identified in our stimulus reading. If she has more than three weeks, it must be from applying half of last’s years unsaved time, which must have been two weeks.
Answer Choice (B) Next year Jennifer will be beginning her fifth year, and we are only told about the vacation time of employees of between one and four years.
Answer Choice (C) We haven’t been told anything about what employees other than Jennifer are doing.
Answer Choice (D) Then we would expect her to have four and a half weeks this year
Answer Choice (E) We have no justification to assume this must be true.
This is a strengthening question, as we are asked: Which one of the following principles, if accepted, would contribute most to Shanna’s defense of her position against that of Jorge?
This stimulus is a dialogue, and our job is to specifically strengthen Shanna’s position against Jorge’s argument. Shanna argues that just by owning a piece of art, you have the right to destroy it should you want to. If we think about it, I’m sure many of us would disagree with Shanna. But our job is to support this position; if it’s yours, you can destroy it.
Jorge responds by arguing that although ownership gives you the right to possess an unique piece of art, you don’t have the right to destroy it. This is because a unique piece of art belongs to posterity (basically, humanity as a whole including those who don’t even exist yet), and so it must be preserved regardless of what you want. Ok, so Jorge thinks the value of unique art to humanity as a whole outweighs the individual rights of the owner. We should look for a principle that empowers the owner. Let’s see what we get:
Answer Choice (A) Well according to Shanna they would still be in their rights to destroy a great piece of art if they owned it and caring for it was inconvenient! But it should be obvious this doesn’t help our position at all.
Answer Choice (B) This principle supports Jorge!
Answer Choice (C) This is arguing that we shouldn’t destroy art because of its value to humanity - that’s Jorge’s position and not Shanna’s!
Answer Choice (D) This is undermining the rights of legal owners, which is the opposite of what we want to do.
Correct Answer Choice (E) The qualification about health and safety might throw you off but this is correct. If individuals can do what they want with their property so long as it doesn’t physically harm others, then they are allowed to destroy artwork they own.
This is a weakening question, since the stem says: Which one of the following, if true, most seriously weakens the argument?
The first thing we learn is that the fines for environmentally damaging accidents are so high that it is cheaper to prevent an accident than it is to deal with the results. The second sentence begins with the conclusion indicator therefore, and concludes that businesses will now invest in preventing accidents. This sentence has a little clause in between commas with the support indicator since, which introduces the final piece of support which is that businesses care about money. It makes intuitive sense, if accidents are going to lose you a lot of money in fines, more than preventing them will, then we would expect those that care about money to prefer prevention to fines. Our job is to weaken this prediction about what businesses are going to do. Let’s see the answers:
Correct Answer Choice (A) Bingo. The argument depends on assuming that businesses will choose to invest in prevention rather than just hoping there won’t be an accident. If they significantly underestimate how likely accidents are, well then they are likely going to assume they don’t have to worry about accidents and the fines, and therefore won’t invest in prevention.
Answer Choice (B) This strengthens the argument, since the prevention of future accidents is a long-term strategy.
Answer Choice (C) This is totally compatible with the argument, the whole point is that businesses are going to invest in prevention because it makes business sense.
Answer Choice (D) This might be appealing if you infer that considering something an ordinary business expense means you don’t mind paying it; the problem is that we’ve been told that businesses care about their profits, and paying fines is more expensive than preventing them. So even if they consider fines an ordinary expense, the argument can still conclude they will choose to pay the cost of prevention instead.
Answer Choice (E) This does nothing to change the fact that prevention is cheaper than fines, and businesses want to maximize their overall profits.
This is a weakening question, indicated in the question stem by: Which one of the following, if accepted by Clay Moltz, would require him to reconsider his conclusion. Bit of a weird question stem, but we should recognize that if introducing something forces him to reconsider his conclusion, then it is weakening his argument.
Our stimulus begins with a bunch of context about a mathematical model which tries to predict the chances of extraterrestrial life existing. To be honest, the model seems to kind of suck. It relies on some pretty large assumptions about how life could exist and whether other star systems resemble ours. It is important for us to remember that we are here to weaken Clay Moltz’ reasoning, not the model’s. It seems Moltz also thinks this model sucks, but his reasoning is itself pretty poor. He infers that because we have not detected any planets outside the solar system, there must not be any life out there. It is again important to recognize that Moltz specifically restricts his conclusion to life as we know it. If some really weird stuff exists out there, that doesn’t contradict his conclusion unless it is life similar to that of Earth. Moltz argument is a classic case of confusing an absence of evidence for evidence of an absence. We can’t infer that just because we haven’t detected any planets, that we know whether there are any out there. Let’s see how the correct answer undermines this argument:
Answer Choice (A) Remember that Moltz only concludes about life as we know it; this answer choice would not force him to reconsider this conclusion.
Answer Choice (B) This could be true and Moltz’ argument would be fine; in fact, he seems to deny that there are planets out there, and therefore this answer is what we would expect to be true if his conclusion is.
Correct Answer Choice (C) This gives an alternative explanation for our not having detected any planets that is much more appealing than Moltz’s ‘there are none to detect’ hypothesis. Of course, we wouldn’t have detected any if we can’t detect any!
Answer Choice (D) This answer might be appealing if you forgot what it is we are supposed to weaken. Our target is Moltz, not the model and its large assumptions.
Answer Choice (E) Alright, but that wouldn’t explain why we haven’t detected any planets. Moltz could accept this without it posing any challenge to his support or conclusion.